User avatar
 
ManhattanPrepLSAT1
Thanks Received: 1909
Atticus Finch
Atticus Finch
 
Posts: 2851
Joined: October 07th, 2009
 
 
 

Re: Q6 - Mayor: Local antitobacco activists

by ManhattanPrepLSAT1 Fri Dec 31, 1999 8:00 pm

I think the best way to imagine this argument at work is to switch the product to gas for your car.

Suppose you live in a small town and the city decides to impose a tax on gas for your car. The city adds $1 per gallon in the hope of raising money to help the city. Now it costs you $4 per gallon, let's say to buy gas in your town, but only $3 per gallon in a town next door. Would you drive the extra 5 miles to get cheaper gas? Maybe, especially if the drive wasn't too far or if the savings were substantial enough.

The same sort of thing is at play here. The conclusion of the argument is that "the taxes themselves would have the sought-after reduction in smoking." Answer choice (C) weakens the argument by suggesting that rather than quitting smoking, people drove to the neighboring city to make their purchase of tobacco.

(A) may strengthen the argument. It says that under some conditions a tax would have the desired effect.
(B) is the best wrong answer. It makes it sound like people will continue to buy tobacco regardless. However, just because they're more likely to continue buying tobacco, does not mean that there was not a reduction in the amount of tobacco consumed.
(C) weakens the argument by showing that the tax would not reduce the amount of tobacco consumed, just shift the purchase to the neighboring town.
(D) is irrelevant. Just because people are more likely to quit when they're well educated about the hazards of smoking does not mean that they wouldn't quit because of a tax imposed on tobacco.
(E) is irrelevant. The conclusion is about whether the tax induces people to quit smoking. The answer choice is about an educational program to encourage people to quit smoking.


#officialexplanation
 
nzheng
Thanks Received: 0
Forum Guests
 
Posts: 10
Joined: May 18th, 2010
 
 
 

Q6 - Mayor: Local antitobacco activists

by nzheng Fri May 21, 2010 10:52 pm

Hi , I am failing to see how C is the choice that will most weaken the argument. I picked B, thinking that it would weaken that premise that taxes themselves would produce a reduction in smoking. Am i missing something here? Thanks.
 
PRADEEPCHANDY
Thanks Received: 0
Forum Guests
 
Posts: 13
Joined: June 27th, 2010
 
 
 

Re: PT52 S3 Q6 Local antitobacco..

by PRADEEPCHANDY Wed Jul 14, 2010 10:08 pm

Here is how I reasoned

There are two possible options to reduce smoking

option 1 : Expanded Antismoking programs --> reduced smoking in city

Option 2 Increased taxes ---> reduced smoking in city

Mayor believes option 2 is the reason and that option 1 is debatable

To weaken the Mayor conclusion

The correct answer must prove option 2 to be false or say option 1 is true

Lets look at options
-----------------------------------------------------------------------------
A actually supports mayor conclusion

B in some ways is supporting mayor conclusion

C : Has potential to prove option 2 to be false - only if we assume that its similar to gas and that people go outside the city to buy tobacco

D. I chose this as the answer because it proves option 1 to be true , if we assume that people will be well informed due to antismoking programs

E. Again in some ways supports the mayor argument


-----------------------------------------

My question: I have noticed in many questions when I am stuck with two options - in the above case between C and D

The way I resolve it is usually by noting the assumption to make each one correct and then keeping the more likely assumption for the correct answer

In this case I chose D because I felt that the its more likely that people will be well informed due to Anti smoking programs than drive to another city to buy tobacco

How do I resolve this and is my approach correct?
 
perrin.samuels
Thanks Received: 1
Forum Guests
 
Posts: 4
Joined: August 05th, 2010
 
 
 

Re: PT52 S3 Q6 Local antitobacco..

by perrin.samuels Thu Aug 05, 2010 8:00 pm

While I agree (B) is the best wrong answer, I'm not convinced a good reason for dismissing it has been adequately articulated. After all, both (B) and (C) make it sound like people will continue to buy tobacco regardless. To say that this does not mean that there was not a reduction in the amount of tobacco consumed doesn't quite hit the nail on the head, IMHO. Lemme take a stab...

We can infer from the survey cited in the argument that there is a connection between dropped cigarette sales and a reduction of smoking in cities which impose stiff tax increases. Answer choice (C) undermines this reasoning by suggesting people will simply buy their cigarettes out of town. Answer choice (B), on the other hand, is more vague and fails to address this connection specifically regarding smoking-related behavior.

As an aside, perhaps I'm reading too deeply, but the question stem does not ask us to undermine the conclusion of the argument, only the reasoning therein.
User avatar
 
ManhattanPrepLSAT1
Thanks Received: 1909
Atticus Finch
Atticus Finch
 
Posts: 2851
Joined: October 07th, 2009
 
 
 

Re: PT52 S3 Q6 Local antitobacco..

by ManhattanPrepLSAT1 Fri Aug 06, 2010 6:27 pm

I really like the points you folks are bringing up. Let me put forward the following example that shows that consumers could be more likely to continue buying cigarettes when a price increase is the result of taxes than for some other reason and yet people would overall be inclined towards a reduction in their amount of smoking.

Suppose people have a 40% chance of continuing to buy cigarettes if the price increase is due to taxes and they have a 20% chance of continuing to buy them if the price increase is due to something else. This would be consistent with answer choice (B).

But if they only have a 40% chance of continuing to buy cigarettes after an increase in price due to higher taxes, that's still going to lead to a reduction in the consumption of cigarettes. This would be consistent with the conclusion, rather than undermine it!

Just thought I add a bit more to the conversation!
 
amandeephayer
Thanks Received: 0
Forum Guests
 
Posts: 1
Joined: August 23rd, 2010
 
 
 

Re: PT52 S3 Q6 Local antitobacco..

by amandeephayer Mon Aug 23, 2010 7:16 pm

nzheng Wrote:Hi , I am failing to see how C is the choice that will most weaken the argument. I picked B, thinking that it would weaken that premise that taxes themselves would produce a reduction in smoking. Am i missing something here? Thanks.

nzheng Wrote:Hi , I am failing to see how C is the choice that will most weaken the argument. I picked B, thinking that it would weaken that premise that taxes themselves would produce a reduction in smoking. Am i missing something here? Thanks.



This is the way I see the question.

Argument:
The Mayor argues increasing taxes causes people to smoke less. He cites the reduction in smoking sales in cities which increases taxes. He argues this is true simply because the statistics say it is true.

He also says that we aren’t sure if education programs are effective in reducing smoking.

Couple of things to keep in mind, he isn’t recommending one course of action over another. He is only stating what he believes that is the taxes will defiantly reduce taxes.

He is also not suggesting that education programs are ineffective he just says the evidence is inconclusive.

Flaw:
There is a flaw in his primary argument. His suggestion is that tax increases will definitely reduce smoking levels because the reduction in sales in other towns.

One thing to keep in mind is you must accept the premises as true and the conclusion can be faulty (which goes to your question). So you must accept that when the taxes on cigarettes increases in a town, fewer people buy cigarettes in that town.

But there is a problem, does that necessarily mean that less people are smoking cigarettes. No. Why because the act of purchasing cigarette has nothing to do with actually smoking them.

Ask yourself something, is there a way I could still smoke as many cigarettes before without buying them. Yes there are plenty of ways, I could manufacture my own cigarettes, I could smuggle my cigarettes, and there are a number of other ways to buy cigarettes.

So here is the flaw, it implies that a reduction in cigarette purchases leads to a reduction in cigarette smoking, as I have demonstrated it is possible to smoke cigarettes without ever actually purchasing cigarettes.

Attack: Whatever answer is true will exploit that flaw which I have highlighted: taxes up then sales go down :. Fewer people are smoking cigarettes or smoking at a lower rate

The right answer may, although it is unlikely, address the second point their which suggests that it is inconclusive evidence that education program are effective. It is possible for there to be conclusive evidence somewhere which suggests that educational programs are infact effective in reducing smoking. (Remember because his conclusion was we can’t be definitely sure education programs reduce what ever the right answer will be will some how prove that education programs will defiantly smoking with no plausibility for error).

What the answer will do is provide an explanation which allows for people to keep smoking at the current rate at the same time it will result in low sales in the town for cigarettes.

A: If the city reduces imposed a tax on cigarettes it will substantially reduce the amount of smoking in a city because the tax is burdensome.

Wrong because this will actually strengthen it.

B. Consumers are more likely to continue buying a product if its price increases due to higher taxes than if it increases for some other reason. This is a very attractive wrong answer.

This actually doesn’t weaken the argument. It actually has no effect on the argument. If this statement was true and the premises above are true (remember premises are always true unless otherwise stated). Then there is no impact on the argument. It transforms the argument into this:

Although people are more likely to keep buying a product at the same rate if taxes increase than if the prices were increases by other means, in this case people smoked fewer cigarettes because of the increase in taxes.

Wrong has no impact on the conclusion.

C. Remember what I said above, could there be another reason for the decline in sales. In fact there is another reason that reason being people simply went to the surrounding towns to purchase the cigarettes. If they went to the surrounding towns to purchase the cigarettes than they aren’t actually smoking less they are just not purchasing in that very town.

This looks good so we keep it.

D. This is actually a very attractive answer as well, as it fits very nicely into weakening the secondary argument.

However, their nothing conclusive about how education programs will increase the awareness. Maybe education programs are in effective in telling people of the harmful effects of cigarettes and it is other factors which increase people’s awareness. Maybe seeing a loved one die from the harmful effects of cigarette smoking is better at informing someone of the harmful effects of smoking than an educational program. Therefore their nothing conclusive about the educational program.

Therefore this doesn’t really weaken the argument that much. It does weaken it a bit but not much. We will keep it.

E. Irrelevant, so what if anti-smoking campaigns lose funding. That isn’t important to this question.

The best answer is C.
 
mrudula_2005
Thanks Received: 21
Forum Guests
 
Posts: 136
Joined: July 29th, 2010
 
 
trophy
First Responder
 

Re: PT52 S3 Q6 Local antitobacco..

by mrudula_2005 Mon Sep 27, 2010 10:45 am

Interestingly enough, I found (E) to be the best wrong answer. This is my thought process: if, as (E) states, the anti smoking education programs that are funded by taxes on cigarettes will tend to lose their funding if they are successful, then the cigarette sales drop in cities with the stiff tax increases (as reported in the stimulus), which is perceived as a success, will actually, according to (E), make the anti smoking education programs lose their funding and thereby will eliminate the tax on cigarettes (since the taxes on cigarettes were meant to fund only the antismoking education programs) - the very tax that was responsible for reduced smoking and then THAT will not at all in the long-run "produce the sought-after reduction in smoking."

I realize that my reasoning assumes that the taxes do reduce smoking and are successful to begin with (which the conclusion is about, I know), but what i'm saying is that with (E) true, the long-term effect would be counter to the conclusion of this stimulus - there would be no "sought-after reduction in smoking" in the end.

What do you think?

Thanks!
User avatar
 
geverett
Thanks Received: 79
Atticus Finch
Atticus Finch
 
Posts: 207
Joined: January 29th, 2011
 
 
 

Re: Q6 - Local antitobacco..

by geverett Wed Sep 14, 2011 11:00 am

Matt,
Just because there is a reduction in the likelihood of an event occurring does that mean there is a reduction in that event occurring? No right? Would answer choice B have weakened the argument if it had just said:

"Consumers are more likely to continue buying a product if it's price increases due to higher taxes." Does more or less likely mean there is necessarily an increase in something happening or a decrease in it's happening? Even if it does not, does this notion of likelihood still meet the lower standards of strengthen/weaken.

thoughts?
 
timsportschuetz
Thanks Received: 45
Elle Woods
Elle Woods
 
Posts: 95
Joined: June 30th, 2013
 
 
trophy
First Responder
 

Re: Q6 - Mayor: Local antitobacco activists

by timsportschuetz Sat Nov 16, 2013 1:17 am

Answer (B) is wrong for the following reason (I completely disagree with the above posters): The argument talks about tax increases affecting the habits of smokers! Answer (B) tries to trap test takers with the phrase "price increases for some other reason" by prodding them to infer that these "other reasons" are the tax increases due to funding of antismoking campaigns. However, the argument specifically states that TAXES would be increased, NOT OTHER items! I could just as easily make the unwarranted assumption that answer choice (B)'s "other reasons" refers to Colombian farmers' unhappiness due to their wives' gaining weight, results in reduced tobacco output worldwide, and therefore resulting in higher prices! DON'T MAKE UNSTATED ASSUMPTIONS FROM ANSWER CHOICES!
 
nja21
Thanks Received: 0
Vinny Gambini
Vinny Gambini
 
Posts: 10
Joined: July 12th, 2014
 
 
 

Re: Q6 - Mayor: Local antitobacco activists

by nja21 Fri Aug 08, 2014 8:20 pm

The reason why (B) does not undermine the argument is that people can continue to buy cigarettes but they can decrease the frequency of it: Instead of buying 4 packs of cigarettes a week I'll buy 2 packs, but I will continue to buy cigarettes. Continuing to buy does not necessarily mean not decreasing. Therefore (B) does not undermine the argument.
 
dfeit10
Thanks Received: 0
Vinny Gambini
Vinny Gambini
 
Posts: 1
Joined: August 07th, 2016
 
 
 

Re: Q6 - Mayor: Local antitobacco activists

by dfeit10 Wed Aug 10, 2016 5:36 pm

But, if the conclusion is that "Cigarette sales drop substantially in CITIES that impose stiff tax increases...", then isn't AC (C) actually strengthening the conclusion? If cigarette sales are increasing in areas surrounding the city, wouldn't that mean that cigarette sales DID decrease in the city with the tariff?

Any help would be greatly appreciated!
 
VictorH605
Thanks Received: 0
Vinny Gambini
Vinny Gambini
 
Posts: 1
Joined: August 28th, 2017
 
 
 

Re: Q6 - Mayor: Local antitobacco activists

by VictorH605 Mon Aug 28, 2017 4:06 pm

dfeit10 Wrote:But, if the conclusion is that "Cigarette sales drop substantially in CITIES that impose stiff tax increases...", then isn't AC (C) actually strengthening the conclusion? If cigarette sales are increasing in areas surrounding the city, wouldn't that mean that cigarette sales DID decrease in the city with the tariff?

Any help would be greatly appreciated!


I agree with this. C being the correct answer does not make sense, because the conclusion states that "cigarette sales drop substantially in cities that impose tax increases on cigarettes."

Meanwhile, if we are to assume that people move out of their current tax-imposed city to a neighbouring city to buy cigarettes, then C does indeed strengthen the argument, and does not undermine it.
 
t-haga-k42
Thanks Received: 0
Vinny Gambini
Vinny Gambini
 
Posts: 8
Joined: March 31st, 2017
 
 
 

Re: Q6 - Mayor: Local antitobacco activists

by t-haga-k42 Wed Nov 22, 2017 9:15 am

Why would it be wrong to assume the Mayor's scope of argument is in the city?
I understand that the conclusion does not specifically say "city", however, the supporting analogous example the mayor uses seems as if he is specifically arguing for the cigarette sales in his city.

Thank you very much in advance for your support.
 
AnnaC659
Thanks Received: 3
Jackie Chiles
Jackie Chiles
 
Posts: 40
Joined: January 03rd, 2018
 
 
 

Re: Q6 - Mayor: Local antitobacco activists

by AnnaC659 Thu Mar 01, 2018 5:30 am

I picked C as an answer but I still have a problem with it that was not mentioned earlier in this thread.

C states "Usually, cigarette sales will incresase substantially in the areas surounding a city after that city imposes stiff taxes on cigarettes."

If increase in cigarette sales in the surrounding ares is caused by people of the Mayor's city travelling to buy them, then yes C would undermine the reasoning in the argument. Nonetheless, isn't this our assumption too? I mean, although we accept that there is a tendency of increase in sale in surrounding ares of the city that increased cigarette tax, this doesn't suggest anything about the reason why such tendency exists. Perhaps the tendency was a mere coincidence, or more people in the surrounding areas started smoking after hearing that they have relatively cheap cigarette!

Can someone explain why C is a legitimate answer not involving a wrongful assumption?
User avatar
 
ohthatpatrick
Thanks Received: 3805
Atticus Finch
Atticus Finch
 
Posts: 4661
Joined: April 01st, 2011
 
This post thanked 1 time.
 
 

Re: Q6 - Mayor: Local antitobacco activists

by ohthatpatrick Fri Mar 02, 2018 8:22 pm

The correct answers to Strengthen and Weaken are miles away from proving or refuting the argument.

Plenty of doubt and room to debate always remains, even with the correct answer in place.

So you're just holding this question to the wrong standards. We're picking this answer because it does more to make us distrust the argument then any other answer does.

That's our standard for any question phrased:
which most undermines (weaken)
which most supports (strengthen)
which is most supported (inference)
which most helps to explain (explain/resolve)

This answer SUGGESTS a plausible alternate explanation for why smoking sales go down in areas that jack up the prices.